Practice Management Flashcards

1
Q

A 37-year-old woman comes to the office for evaluation for cosmetic rhinoplasty. The patient repeatedly reports to the surgeon that she “is ugly” and “can’t stand” her nose. Physical examination shows no abnormalities consistent with her concerns. Medical history includes five visits to different plastic surgeons in the past 4 months and each surgeon has declined to perform surgery. The surgeon refuses to perform the surgery and refers the patient to a health psychologist. The surgeon’s decision is a demonstration of which of the following principles?

A) Beneficence
B) Iniquity
C) Justice
D) Maleficence
E) Noxiousness

A

The correct response is Option A.

In refusing to operate on a patient with body dysmorphic disorder and instead referring the patient to a health psychologist, the surgeon is practicing the ethical principle of beneficence—the doing of good. In cases of medical ethics, the terms maleficence, iniquity, and noxiousness are all associated with doing harm. In this scenario, the surgeon is clearly not doing harm. Justice, the ethical principle requiring doctors to ensure that medical care is available to all, is not applicable to this scenario.

2018

How well did you know this?
1
Not at all
2
3
4
5
Perfectly
2
Q

Which of the following items are required for the surgeon to perform along with the nurse and anesthesiologist on the World Health Organization Surgical Safety Checklist before the induction of anesthesia?

A) Confirmation of instrument, sponge, and needle counts
B) Confirmation of patient identity, surgical site, procedure, and consent
C) Discussion of specimen labeling
D) Discussion of whether essential imaging needs to be displayed
E) Discussion of whether there were any equipment problems during case

A

The correct response is Option B.

Confirmation of patient identity, surgical site, procedure, and consent is the only item out of the above answer choices that is required before the induction of anesthesia. The discussion of imaging display occurs after anesthesia induction and before skin incision. Confirmation of instrument, sponge, and needle counts; specimen labeling; and equipment problems are not appropriate before induction of anesthesia, and are performed before the patient leaves the operating room.

2018

How well did you know this?
1
Not at all
2
3
4
5
Perfectly
3
Q

Which of the following is a Type I error?

A) Accepting the alternate hypothesis when it is actually true
B) Accepting the null hypothesis when it is actually false
C) Accepting the null hypothesis when it is actually true
D) Rejecting the alternate hypothesis when it is actually true
E) Rejecting the null hypothesis when it is actually true

A

The correct response is Option E.

A Type 1 error is the error of rejecting the null hypothesis when it is actually true. A Type 2 error is the error of not rejecting the null hypothesis when it is actually false. All the other answers are not correct.

2018

How well did you know this?
1
Not at all
2
3
4
5
Perfectly
4
Q

After successfully completing training at an accredited United States or Canadian training institution, the maximum amount of time a surgeon is allowed to advertise as board eligible while seeking initial certification with the American Board of Plastic Surgery (ABPS) is which of the following?

A) 1 year
B) 3 years
C) 8 years
D) 10 years
E) There is no limit

A

The correct response is Option C.

The American Board of Medical Specialties (ABMS) has held an effective official Board Eligibility Policy since 2012. The policy recognizes physicians’ need to advertise with the term “Board Eligible” during their preparatory time for initial board certification, but closes off the potential for abuse through the use of the term indefinitely. This period of time generally varies between ABMS Member Boards as 3 to 7 years following successful completion of accredited training, plus any additional practice time as required by the Member Board for admissibility to their certifying examination. The American Board of Plastic Surgery (ABPS) is one of approximately 24 Member Boards of the ABMS. For the ABPS, the time period is restricted to a maximum of 8 years (7 years plus an additional 1 year allowance to meet the necessary practice requirement to successfully complete both written and oral examinations in plastic surgery).

Exceptions are considered in instances of military deployment, acute illness, or other individual circumstances according to review by the Member Board.

Once certified, diplomates of the ABPS must complete professionalism requirements, self assessment activities, practice improvement activities, and an examination every 10 years as required by the Maintenance of Certification Program in order to maintain their certification status.

2018

How well did you know this?
1
Not at all
2
3
4
5
Perfectly
5
Q

A surgeon is approached by a charitable organization requesting support for a silent auction fund-raiser. Which of the following services can be donated according to the American Society of Plastic Surgeons (ASPS) code of ethics?

A) Abdominoplasty
B) Augmentation mammaplasty
C) Botulinum toxin A for forehead rejuvenation
D) Rhytidectomy
E) Single-site liposuction

A

The correct response is Option C.

The American Society of Plastic Surgeons (ASPS) has guidelines and policies governing ethical behavior of plastic surgeons. Participation in charitable events is permitted, but must be done in a manner that does not provide implicit understanding of a performance of a procedure for which the patient has not been medically evaluated. Furthermore, an implied or real financial incentive to have a procedure is considered unethical. Botulinum toxin A for forehead rejuvenation is most appropriate. All other options are specific in regards to treatment, for which the patient has ostensibly purchased through the silent auction, but may not be a candidate. Offering free implants also implies that an augmentation mammaplasty is to be done, which is implicitly offering the procedure. No procedures requiring an incision are allowed.

2018

How well did you know this?
1
Not at all
2
3
4
5
Perfectly
6
Q

A recent research manuscript is reviewed at a plastic surgery journal club. The article is a case-control study examining the outcomes following peripheral nerve decompression. Which of the following is the level of evidence that this study provides?

A) Level I
B) Level II
C) Level III
D) Level IV
E) Level V

A

The correct response is Option C.

Level I data are defined as high-quality prospective cohort studies with adequate power or systematic review of these studies. Level II evidence represents lesser quality prospective cohorts, retrospective cohort studies, untreated control from an RCT, or systematic review of these studies. Level III evidence is correct and represents case control studies or systematic review of these studies. Level IV evidence is case series studies. Level V evidence is expert opinion, case report, or clinical example; or evidence based on physiology, bench research, or “first principles.”

2018

How well did you know this?
1
Not at all
2
3
4
5
Perfectly
7
Q

As of January 1, 2018, surgeons enrolled in Centers for Medicare & Medicaid Services (CMS) Electronic Health Record (EHR) Meaningful Use incentive programs are subject to which of the following payments?

A) No payments, enrollment is mandatory
B) 1% Penalty adjustments below existing Medicare/Medicaid reimbursements
C) Tax credits commensurate with level of participation
D) 1 To 3% positive adjustments above existing Medicare/Medicaid reimbursements
E) Up to $44,000 to offset EHR initiation costs

A

The correct response is Option B.

The concept of “meaningful use” of electronic health records was introduced by the Health Information Technology for Economic and Clinical Health (HITECH) Act. The HITECH Act was one of many measures enacted by the American Reinvestment and Recovery Act of 2009 to modernize the United States’ infrastructure. The Centers for Medicare & Medicaid Services (CMS) and the Office of the National Coordinator for Health Information Technology (ONC) subsequently published a series of specific rules for health care providers under Electronic Health Record (EHR) Incentive Programs to support the concept of meaningful utilization of electronic health records espoused by HITECH. Both Medicare and Medicaid have related but independent criteria for demonstrating meaningful use through their respective EHR Incentive Programs.

In general, “meaningful use” is defined by the use of certified EHR technology in a meaningful manner (eg, electronic prescribing); ensuring that the certified EHR technology is connected in a manner that provides for the electronic exchange of health information to improve the quality of care; and that in using certified EHR technology, the provider must submit to the Secretary of Health & Human Services (HHS) information on quality of care and other measures.

While maximum EHR benefits through the Medicare Incentive Program initially totaled up to $44,000 if specific criteria were met over 5 consecutive years, participants must have begun meeting incentive program criteria by 2014, and the last year these incentive payments were paid was 2016.

Participation for both Medicare and Medicaid EHR Incentive Programs is currently completely “voluntary.” However, since 2015, failure to participate in the incentive programs including attestation to your meaningful use of certified EHR technology on a yearly basis has resulted in a penalty in the form of negative adjustments to Medicare/Medicaid reimbursement rates of at least 1% from CMS. On its own, EHR Meaningful Use “incentive program” has transformed into a euphemism for a de facto penalty program.

2018

How well did you know this?
1
Not at all
2
3
4
5
Perfectly
8
Q

A 20-year-old woman comes to the office with her parents because of her significant concern with the appearance of a scar on her forehead of 1 year’s duration. Physical examination shows a well-healed scar that blends in nicely with the surrounding skin and is difficult to see at conversation distance. The patient’s parents do not see the need for any intervention since they also find the scar difficult to see. Which of the following must be present to confirm a diagnosis of body dysmorphic disorder in this patient?

A) History of treatment for an eating disorder
B) Occasional social anxiety
C) Preoccupation with obvious flaws in her appearance
D) Prior rhinoplasty
E) Repetitive behavior related to her appearance concerns

A

The correct response is Option E.

According to the Diagnostic and Statistical Manual of Mental Disorders, 5th edition, (DSM-V) the following criteria define the diagnosis of body dysmorphic disorder (BDD):

Preoccupation with one or more perceived defects or flaws in physical appearance that are not observable or appear slight to others.

At some point during the course of the disorder, the individual has performed repetitive behaviors (e.g., mirror checking, excessive grooming, skin picking, reassurance seeking) or mental acts (e.g., comparing his or her appearance with that of others) in response to the appearance concerns.

The preoccupation causes clinically significant distress and impairment in daily function.

An eating disorder may be seen with body dysmorphia, but is not pivotal to its diagnosis.

A diagnostic criterion describing repetitive behaviors or mental acts in response to preoccupations with perceived defects or flaws in physical appearance has been added since the DSM-IV-TR, consistent with data indicating the prevalence and importance of this symptom.

Prior history of cosmetic surgery (rhinoplasty in this case) is not part of the diagnostic criteria for body dysmorphic disorder.

2017

How well did you know this?
1
Not at all
2
3
4
5
Perfectly
9
Q

A 50-year-old man recently enrolled in a new health insurance plan under the Affordable Care Act. Which of the following benefits is NOT offered as a basic service to this patient?

A) Dental care
B) Mental health and substance use treatments
C) Prescription drugs
D) Preventative services
E) Rehabilitation services

A

The correct response is Option A.

The Affordable Care Act (ACA) signed into law in 2010 began enrollment in 2013. One of the most fundamental components of the ACA was that any new health insurance plan must offer ten “Essential Health Benefits.” These include: 1) outpatient care; 2) emergency room visits; 3) hospitalization; 4) maternity and newborn care; 5) mental health and substance use treatments; 6) prescription drugs; 7) rehabilitation and rehabilitative services and devices; 8) laboratory tests; 9) preventative services and chronic disease care; and 10) pediatric services including dental and vision. Because the individual in the vignette is 50 years old, his new plan will not cover basic dental services.

2017

How well did you know this?
1
Not at all
2
3
4
5
Perfectly
10
Q

The plastic surgeon is evaluating the strength of a recently published clinical study. The surgeon notes that the study does not include a power calculation to assess the adequacy of the sample size. Which of the following study errors is minimized by the inclusion of a statistical power calculation?

A) Type I
B) Type II
C) Type III
D) Type IV

A

The correct response is Option B.

The statistical power is the probability that a sample can detect a treatment effect if a true effect exists. It is the value defined by the equation 1??, where ? represents the probability of making a type II error. The alpha level represents the probability of making a type I error. Statistical power increases with increasing sample size. The power of a study is meaningful only when it is associated with detecting a specific clinically meaningful treatment effect or difference that is likely to occur, and thus a power calculation requires the researcher to specify the desired alpha level, sample size, and magnitude of the clinical meaningful treatment effect.

Type I error involves rejecting the null hypothesis and falsely concluding that there is a treatment effect. Researchers often guard against this type of error by setting a low significance (α) level.

Type II error involves failing to reject the null hypothesis and falsely concluding that there is a treatment effect. Setting an adequate sample size to increase the statistical power of the study minimizes this.

Type III and type IV errors are more recently described errors that may be committed by the investigator and focus on the two main parts of the reasoning chain: the model used to evaluate the question and the evaluation and conclusion of the study. Type III error involves correctly rejecting the null hypothesis for the wrong reason. Type IV error involves the incorrect interpretation of a correctly rejected hypothesis.

2017

How well did you know this?
1
Not at all
2
3
4
5
Perfectly
11
Q

According to HIPAA regulations, which of the following is considered individually identifiable personal health information?

A) Age
B) Date of birth
C) Gender
D) Race
E) State of residence

A

The correct response is Option B.

According to HIPAA Privacy Rules, “Individually identifiable health information” is information, including demographic data, that relates to:

the individual’s past, present, or future physical or mental health or condition

the provision of health care to the individual, or

the past, present, or future payment for the provision of health care to the individual

and that identifies the individual or for which there is a reasonable basis to believe can be used to identify the individual. Individually identifiable health information includes many common identifiers (e.g., name, address, birth date, social security number).

2017

How well did you know this?
1
Not at all
2
3
4
5
Perfectly
12
Q

A university-based plastic surgery division begins planning to open an offsite ambulatory surgery center (ASC). The university is located in a state that requires a certificate of need before building an ASC. Which of the following best describes the reason for requiring a certificate of need for this project?

A) To control health care ASC costs and allow coordinated planning of new services and construction
B) To demonstrate that the ASC qualifies for Medicare funding
C) To ensure that nurses working at the ASC abide by the ethical principle of “do no harm”
D) To ensure that physicians working at the ASC abide by the ethical principle of “do no harm”
E) To require that at least one registered nurse is on-site at the ASC during surgeries

A

The correct response is Option A.

According to the National Conference of State Legislatures:

Certificate of need (CON) programs are aimed at restraining health care facility costs and allowing coordinated planning of new services and construction. Laws authorizing such programs are one mechanism by which state governments seek to reduce overall health and medical costs. Many CON laws initially were put into effect across the nation as part of the federal Health Planning and Resources Development Act of 1974. Despite numerous changes in the past 30 years, about 36 states retain some type of CON program, law, or agency as of 2014.

2017

How well did you know this?
1
Not at all
2
3
4
5
Perfectly
13
Q

In the evaluation of evidence-based medicine, which of the following is considered Level V evidence?

A) Case series
B) Individual case report
C) Prospective cohort study
D) Randomized clinical trial
E) Retrospective comparative study

A

The correct response is Option B.

Evidence-based medicine, particularly randomized controlled trials, influences many daily decisions within the medical specialties. The evidence-rating scale for therapeutic studies is as follows:

LevelDescription

IHigh-quality, multicentered or single-centered, randomized controlled trial with adequate power; or systematic review of these studies

IILesser-quality, randomized controlled trial; prospective cohort study; or systematic review of these studies

IIIRetrospective comparative study; case-control study; or systematic review of these studies

IVCase series

VExpert opinion; case report or clinical example; or evidence based on physiology, bench research, or “first principles”

2017

How well did you know this?
1
Not at all
2
3
4
5
Perfectly
14
Q

For the purpose of billing evaluation and management (E/M) services, a “new patient” is considered one who has not received any professional services from another plastic surgeon in a group practice within a minimum of which of the following months?

A) 6
B) 12
C) 18
D) 24
E) 36

A

The correct response is Option E.

According to the 2015 American Medical Association’s (AMA) CPT book, “… a new patient is one who has not received any professional services from the physician/qualified health care professional or another physician/qualified health professional of the exact same specialty and subspecialty who belongs to the same group practice, within the past three years.” If a patient has received professional services evaluation and management (E/M) or other face-to-face service – (such as surgical procedures), within the past three years, from the physician/qualified health care professional, or another physician/qualified health care professional of the exact same specialty and subspecialty who belongs to the same group practice, he or she would be defined as an “established patient.”

The definition of a “new patient” by the Centers for Medicare and Medicaid Services differs slightly from the AMA’s. Instead of including physicians in the same specialty and subspecialty, for Medicare E/M services, the same specialty is determined by the physician’s or practitioner’s primary specialty enrollment in Medicare. The three-year time period still applies.

2017

How well did you know this?
1
Not at all
2
3
4
5
Perfectly
15
Q

A 72-year-old man with type 2 diabetes mellitus is evaluated because of a 2-month history of a plantar heel ulcer. Routine standard therapies have been unsuccessful. Administration of a new topical growth factor ointment indicated for refractory diabetic foot ulcers is planned. The best literature supporting the use of this product for this patient describes a single non-blinded, prospective, randomized controlled trial with low patient numbers and poor long-term follow-up. Which of the following is the level of evidence associated with the therapeutic use of this product?

A) Level I
B) Level II
C) Level III
D) Level IV
E) Level V

A

The correct response is Option B.

The practice of evidence-based medicine involves the interpretation of the best available evidence in order to make informed clinical decisions regarding the care and treatment of patients. Because the quality of studies published in the literature is of varying quality, a hierarchical grading system often described as the levels of evidence has been employed by multiple different specialties.

Typically, randomized controlled trials (RCTs) are considered the highest level of evidence. However, RCTs cannot be used to answer all types of clinical questions. For example, clinical questions regarding the prognosis of a given condition without any treatment cannot be answered with an RCT. In addition, all RCTs are not of the same quality. Systematic reviews or meta-analysis of similar RCTs can increase the power of the studies. In contrast, RCTs that are not blinded, have poor methods of randomization, lack exclusion criteria, or that have low numbers and are underpowered are considered to be a lower level of evidence. The RCT in this question is non-blinded and underpowered with poor follow up and, therefore, a level II.

The ASPS has published the following evidence rating scale:

2016

How well did you know this?
1
Not at all
2
3
4
5
Perfectly
16
Q

A non-emancipated, 17-year-old girl comes to the office to request bilateral reduction mammaplasty. Which of the following is the most appropriate next step in management?

A) Perform reduction mammaplasty after obtaining consent from both parents
B) Perform reduction mammaplasty after obtaining consent from one parent
C) Perform reduction mammaplasty without parental consent
D) Wait to perform reduction mammaplasty until the patient is 18 years old
E) Wait to perform reduction mammaplasty until the patient is 21 years old

A

The correct response is Option B.

The next most appropriate step in management is to perform the reduction mammaplasty with parental consent from one parent. Almost all states require patients younger than 18 years of age to have the consent of one parent for a surgical procedure. Emancipated minors younger than 18 years of age are legally able to consent for medical procedures without parental consent. The criteria for an emancipated minor generally include: marriage, military service, financial independence, living apart from the parents, and parenthood.

2016

How well did you know this?
1
Not at all
2
3
4
5
Perfectly
17
Q

A hospital undertakes a root cause analysis to investigate a wrong-site surgery. The first step is to determine if there was compliance with the Universal Protocol mandated by the Joint Commission. Which of the following is a required component as mandated by the Universal Protocol?

A) Insurance pre-authorization for the procedure
B) Preoperative marking of the surgical site
C) Surgical debrief immediately following the procedure
D) Use and documentation of a safe surgery checklist
E) Use of standardized preoperative order templates for common procedures

A

The correct response is Option B.

In 2003, the Joint Commission (formerly, the Joint Commission on Accreditation of Healthcare Organizations or JCAHO) made the elimination of wrong-site surgeries a national patient safety goal and the following year required compliance with the Universal Protocol for Preventing Wrong Site, Wrong Procedure, and Wrong Person Surgery. The Universal Protocol requires three separate steps:

The proper preoperative identification of the patient by the three members of the team (surgeon, anesthesiologist, and nurse)

Preoperative marking of the surgical site

A final “time out” just prior to the surgery or procedure regardless of where it is performed

Use and documentation of a safe surgery checklist is a new quality reporting program implemented by the Centers for Medicare and Medicaid Services (CMS) for ambulatory surgery centers and became part of the payment determination in 2015. Although this has been used to help fulfill the requirements of the Universal Protocol, it is not a required component of the Universal Protocol.

Incorporating a debrief immediately following a surgical procedure is a typical hospital policy requirement and good medical practice, but will not prevent wrong site, wrong procedure, or wrong person surgery. It is not a required component of the Joint Commission’s Universal Protocol.

Use of standardized preoperative order templates and insurance pre-authorization is not a required element of the Universal Protocol.

2016

How well did you know this?
1
Not at all
2
3
4
5
Perfectly
18
Q

A physician wants to research the effectiveness of a new pain medication with his patients. He plans a prospective randomized study with an experimental group that will receive the new pain medication and a control group that will receive a placebo. Data from each group will be reported and analyzed for the significance of pain relief from the new medication. Which of the following statistical tests is most likely to determine the appropriate sample size for this study?

A) ANOVA
B) Chi-square
C) Fisher exact test
D) Statistical power
E) Student t-test

A

The correct response is Option D.

Statistical power is the likelihood of identifying a predefined clinical significance. Analysis of statistical power explores associations among effect size, sample size, test size, and power of the test. Statistical power can be used to determine sample size or verify a nonsignificant result after data collection. Statistical power of 80% is ideal for any study.

2016

How well did you know this?
1
Not at all
2
3
4
5
Perfectly
19
Q

The concept of “meaningful use” in the context of the electronic medical record refers to which of the following?

A) Compartmentalization of medical records between a patient’s providers, requiring individual release forms for records in compliance with Health Insurance Portability and Accountability Act (HIPAA) regulations
B) Increasing the ability of health care systems to allow small business employers access to patient medical information for calculation of insurance premiums
C) Increasing the ability of health care systems to protect patient medical information in compliance with HIPAA regulations
D) Providing documentation of only vital signs in electronic medical records for each patient seen
E) Using electronic prescriptions when more convenient for the patient

A

The correct response is Option C.

Implementation of electronic medical record (EMR) is one part of updating the United States health care infrastructure. Part of utilizing EMR is the concept of “meaningful use,” which refers to using the approved EMR systems to their full potential in order to provide optimization of health care data documentation to improve the quality of health care. There are five categories that have become priorities in establishing adequate parameters of health care:

Improve quality, safety, efficiency, and reducing health disparities

Engage patients and families in their health

Improve care coordination

Improve population and public health

Ensure adequate privacy and security protection for personal health information

2016

How well did you know this?
1
Not at all
2
3
4
5
Perfectly
20
Q

A 56-year-old woman undergoes reduction mammaplasty. Her primary insurance coverage is provided by Medicare. The surgeon, a contracted provider, submits a bill for . Per the contract with the insurance company, the provider is paid from Medicare and the patient is responsible for a co-pay of . Which of the following statements in regard to the difference between the provider’s bill and his payment is most accurate?

A) The difference can be allocated to bad debt for reasons of accounting and tax preparation
B) The difference can be recouped by the provider by submitting a new bill for local flap rearrangement and complex wound repair of the breasts
C) The difference is a contractual adjustment and the service is considered paid in full
D) The difference should be billed to the patient
E) The difference should be financed on a payment plan

A

The correct response is Option C.

The difference of the bill and the payment is considered a contractual adjustment and the services are paid in full. As a contracted provider for Medicare, the physician agrees to accept payment according to the Medicare fee schedule. Payment comes in two forms: direct payment from Medicare and co-payment from the patient. Many Medicare enrollees also pay for a supplemental health insurance policy. This plan provides payment for deductibles and co-pays that Medicare does not pay. If such insurance is carried, then the patient’s co-payment for services is paid by the supplemental insurance company. The patient’s obligation is the contracted co-payment, whether paid personally by them or by their supplemental policy. Virtually all insurance companies follow these same regulations.

Medicare consists of two basic parts: Part A and Part B. Part A covers hospitalization, skilled nursing facilities, and hospice care. Part B covers provider and related services: doctors, physical therapists, laboratories, durable equipment, mental health, etc. Enrollees do not pay a monthly premium for Part A if Medicare taxes were paid while they were employed and they are age 65 and receive retirement benefits (social security), or if they are under age 65 and receive social security benefits or have end-stage renal disease (with specific sub-requirements). In 2014, Part B carried a monthly premium of .90, and a yearly deductible of . Both fees will not change in 2015.

Based on federal and state laws, it is illegal to balance bill patients for fees greater than the contractual agreement for those services. The difference between the bill and the payments cannot by written off as bad debt. Truly owed money that is not paid is considered bad debt. No further money is owed after all contractual agreed-upon payments have been made. Submitting new bills for local flap rearrangement is considered to be unbundling, since this aspect of the procedure is included in breast reduction surgery and payment. Unbundling is a form of insurance fraud and is subject to significant penalties.

2016

How well did you know this?
1
Not at all
2
3
4
5
Perfectly
21
Q

Since August of 2013, manufacturers of medical devices and pharmaceuticals must report physician payments greater than .00 or annually. Centers for Medicare & Medicaid Services (CMS) has now publicly released this information in compliance with which of the following legislative mandates?

A) Government in Sunshine Act
B) The Health Insurance Portability and Accountability Act
C) The Patient Safety and Quality Improvement Act
D) The Physician Reporting Act
E) The Protecting Access to Medicare Act

A

The correct response is Option A.

The CMS Sunshine Act is also known as Open Payments. It mandates that manufacturers of devices, biologics, drugs, and medical supplies, and reports all physician payments starting on August 1, 2013. Starting March 31, 2014, the manufacturers began reporting. Payments of less than are exempt from reporting. Aggregate annual totals of or more per company must be disclosed. CMS plans to release the reported payment information on a public Web site by September 30, 2014. The Protecting Access to Medicare Act mandates that implementation of ICD-10 does not begin until FY16. The Health Insurance Portability and Accountability Act (HIPAA) of 1996 provides rules about individual health information identifier protection. The Patient Safety and Quality Improvement Act of 2005 is legislation regulating the protection of patient safety work product and enforcement activities. The Physician Reporting Act does not exist.

2016

How well did you know this?
1
Not at all
2
3
4
5
Perfectly
22
Q

A 35-year-old woman is scheduled to undergo low-volume liposuction of the lower abdomen in an office setting. Intravenous sedation, in addition to local and tumescent anesthesia, is planned. Which of the following is essential to have in the operative suite according to the Guidelines for Office-Based Anesthesia?

A) Central venous catheter kit
B) Electrocautery unit
C) Intubation equipment
D) Tracheostomy set
E) Warming blanket

A

The correct response is Option C.

To ensure patient safety during office-based procedures, a system of quality care needs to be established in each facility, emphasizing maintenance of the appropriate facilities, equipment, personnel, protocols, and procedures.

When administering anesthesia of any kind in an office setting, the surgeon should follow the American Society of Anesthesiologists’ “Guidelines for Office-Based Anesthesia,” and “Practice Guidelines for Sedation and Analgesia by Non-Anesthesiologists.”

The facility should be outfitted with the appropriate medical equipment, materials, and drugs necessary to provide anesthesia, recovery ministration, cardiopulmonary resuscitation, and provisions for potential emergencies. Anesthesia equipment should include suctioning apparatus, appropriately sized airway equipment, including laryngoscope blades, means of positive-pressure ventilation, intravenous equipment, pharmacologic antagonists, basic resuscitative medications, and, in the event of deep sedation, defibrillator equipment.

Additional equipment, such as a central venous catheter kit, electrocautery unit, warming blanket, and tracheostomy set may be desirable based on the type of patients and/or cases being performed, but are not considered essential in the case of the patient described.

2015

How well did you know this?
1
Not at all
2
3
4
5
Perfectly
23
Q

Based on the code of ethics of the American Society of Plastic Surgeons, which of the following actions, if committed by a member, can be cause for disciplinary action?

A) The board-eligible surgeon chooses to advertise in an airline magazine
B) The member chooses to place patient testimonials on his or her Web site
C) The member participates in a charity raffle, fund-raising event, contest, or other promotion in which the prize is nonsurgical facial rejuvenation
D) The member uses his or her own before-and-after photos on the Web site
E) The out-of-network member charges wholly disproportionate fees for emergency care

A

The correct response is Option E.

Disciplinary actions can be taken for charging exorbitant fees, particularly of a non-contractual nature (e.g., emergency care). Fees are exorbitant when they are wholly disproportionate to the services rendered. The reasonableness of fees depends upon the novelty and difficulty of the procedures involved, the skill required to provide proper care, the time and labor required, the fee charged for similar services by similarly situated peers, and whether or not the patient had agreed in advance to the fee. Except in instances of emergencies or urgent and life-threatening disease or injury, nothing in this principle shall be construed to prohibit a member from requiring prepayment of professional fees for all elective surgical operations.

Sanctions can be taken if the member participates in a charity raffle, fund-raising event, contest, or other promotion in which the prize is any procedure defined as a medical service that requires an incision. Examples of services that require an incision include, but are not limited to, rhytidectomy, augmentation mammaplasty, blepharoplasty, and liposuction. Examples of medical services that would not be considered procedures include, but are not limited to, injections (botulinum toxin, hyaluronic acid), microdermabrasion, and other skin surface treatments. Nonsurgical means do not fall under this category.

Sanctions can be taken if the Web site contains a testimonial pertaining to the quality and efficacy of medical care if the experience of the endorser does not represent the typical experience of other patients or if, because of the infrequency and/or complexity of such care, results in other cases cannot be predicted with any degree of accuracy. Also, sanctions may be taken if it contains a testimonial or endorsement pertaining to the quality of the member’s medical care or the member’s qualifications if the endorser has been compensated by the member or a third party retained by the member for making such testimonial or endorsement. Honest, well-represented, unbiased, uncoerced testimonials are allowed.

Disciplinary actions can be taken if the Web site contains photographs, images, or facsimiles of persons who falsely or deceptively portray a physical or medical condition, injury, or disease, including obesity, or recovery of relief therefrom; or if it contains photographs, images, or facsimiles of persons who have received the services advertised, but who have experienced results that are not typical of the results obtained by the average patient, without clearly and noticeably disclosing that fact; also, if it contains photographs, images, or facsimiles of persons before and after receiving services, which use different light, poses, or photographic techniques to misrepresent the results achieved by the individual.

2015

How well did you know this?
1
Not at all
2
3
4
5
Perfectly
24
Q

According to the Health Insurance Portability and Accountability Act’s Security Rule, which of the following is the basic requirement for storing patient photographs on a personal computer?

A) De-identification of patient names in files
B) Encryption using an approved process
C) Institutional review board approval
D) Password protection of the personal computer
E) Storage of patient photographs is not permitted on personal devices

A

The correct response is Option B.

According to the U.S. Department of Health and Human Services, “the Health Insurance Portability and Accountability Act (HIPAA) Security Rule establishes national standards to protect individuals’ electronic personal health information (ePHI) that is created, received, used, or maintained by a covered entity. The Security Rule requires appropriate administrative, physical and technical safeguards to ensure the confidentiality, integrity, and security of electronic protected health information.” To this end, HIPAA requires that ePHI be protected using encryption processes that have been tested by the National Institute of Standards and Technology and judged to meet this standard.

Password protection can prevent access to electronic devices but does not allow for data on a device to be deleted in the event of a breach. De-identification of file names does not change the electronic information that is stored within a file and, therefore, is inadequate in protecting ePHI. Institutional review boards are primarily responsible for reviewing and monitoring research that involves human subjects and are generally separate from HIPAA-related regulations.

2015

How well did you know this?
1
Not at all
2
3
4
5
Perfectly
25
Q

Which of the following is a standard for accreditation of an ambulatory surgery facility as it relates to plastic surgeons?

A) All team members of the ambulatory facility caring for pediatric patients must be certified in Pediatric Advanced Life Support
B) Ambulatory facilities are inspected every 5 years
C) Patients undergoing ambulatory surgery during general anesthesia require a responsible adult to monitor them for 1 to 2 hours after discharge from the ambulatory surgery center
D) Plastic surgeons working in the ambulatory surgery facility must be board certified by the American Board of Plastic Surgery
E) Surgeons operating in an ambulatory surgery center are required to demonstrate that they hold unrestricted hospital privileges at an acute-care hospital within 30 minutes’ driving time of the facility

A

The correct response is Option E.

The American Association for Accreditation of Ambulatory Surgery Facilities (AAAASF) requires surgeons to be Board Certified or Board Eligible with a Board recognized by the American Board of Medical Specialties. A patient who underwent general anesthesia needs a responsible adult to supervise him/her for 12 to 24 hours. Surgeons are required to demonstrate that they hold unrestricted hospital privileges at an acute-care hospital within 30 minutes’ driving time of the ambulatory facility. If pediatric patients are cared for, at least one member of the team needs to be certified in Pediatric Advance Life Support (PALS). Ambulatory care facilities are inspected every 3 years by the AAAASF.

2015

26
Q

A 5-year-old girl, who is a Jehovah’s Witness, is scheduled to undergo extensive cranioplasty. Bloodless surgery has been scheduled at an institution offering a blood conservation program. During the procedure, the patient becomes hemodynamically unstable from excessive, ongoing intraoperative bleeding and is found to have a hemoglobin level of 6.0 g/dL. The patient’s parents refuse to consent to a blood transfusion. Which of the following is the most appropriate next step in management?

A) Administration of blood products
B) Administration of erythropoietin
C) Initiation of vasopressors
D) Resuscitation with albumin
E) Resuscitation with hypertonic saline

A

The correct response is Option A.

The issue of religious beliefs and medical management is a difficult ethical scenario that can arise in the care of pediatric patients. It is generally accepted in many countries with the support of international law that a minor should not be put at risk because of the religious beliefs of his/her parents. In this case, a blood transfusion is clearly needed but is not a medical treatment accepted by Jehovah’s Witnesses. In the United States, the American Academy of Pediatrics recommends that in cases of “an imminent threat to a child’s life,” physicians in some cases may “intervene over parental objections.” In general, a contingency of bloodless surgery programs for pediatric patients is the reservation of the legal right to give blood if an emergent need arises, despite the lack of parental consent. It is generally accepted that blood products may be given to minors if it is in the best interest of the patient despite parental religious beliefs or other objection. This course of action has been supported a number of times in the court of law.

2015

27
Q

A 46-year-old woman receives several thousand dollars’ worth of injectable filler and neurotoxin treatments following an initial consultation with the plastic surgeon. Before receiving the injections, she indicated that she understood the planned treatments and was made aware of the costs. She provided informed consent and paid for the services with a credit card. The plastic surgeon calls the patient for follow-up several weeks after the treatment, and the patient reports being happy with the results. Several months later, the plastic surgeon receives a notice from the credit card company that the patient is now disputing the charges, and the credit card company promptly removes the fees from the plastic surgeon’s account. Which of the following is the most appropriate action by the plastic surgeon in response to this situation?

A) Call the patient and advise her that her charges will be turned over to the police for fraud
B) Call the patient who referred her and ask if she could encourage this patient to reverse the charges
C) Contact a lawyer to consider legal action against this patient
D) Ensure that all future patients sign a policy in advance of any treatment that waives their privacy protection under HIPAA in matters of dispute of payment
E) Send documentation to the credit card company of the treatments the patient received and proof that she had signed informed consent

A

The correct response is Option D.

Although it may seem aggressive to have patients sign a financial policy before receiving treatment stating that if there are any matters in dispute after the fact, they waive their right to privacy under the Health Insurance Portability and Accountability Act of 1996 (HIPAA) guidelines, this is the only way in which a case like this may be successfully handled. Without such protection, no information (including the fact that the person was even a patient) may be shared with a credit card company, another patient, the police, or a collection agency because it would be a violation of HIPAA, even if appropriate informed consent were obtained. Calling a patient with any kind of threat about a financial matter should never be done because this would violate ethical standards. Without a patient specifically waiving his/her right to privacy under HIPAA in a signed document, no information about that patient may be shared with the credit card company, and the physician is unable to confirm that the patient is indeed under his care. HIPAA affords complete protection of a patient’s privacy, and, in situations such as these, to the great disadvantage of physicians who accept credit cards for payment. Any dispute cannot be properly addressed by the physician without a patient signing away these rights specifically.

2015

28
Q

A 17-year-old girl comes to the office for evaluation for an abdominoplasty 1 year after the delivery of her first child. Following evaluation, the patient is deemed a good surgical candidate. The patient is married, but she came to the consultation alone. Which of the following is the most appropriate next step?

A) Obtain consent from her husband
B) Obtain consent from one parent
C) Obtain consent from the patient
D) Re-evaluate the patient at 18 years of age
E) Re-evaluate the patient at 21 years of age

A

The correct response is Option C.

The most appropriate next step in management is to perform the abdominoplasty without parental consent. Because the patient is married and has a child, she is an emancipated minor and is legally able to consent to medical procedures. The criteria for an emancipated minor generally include: marriage, military service, financial independence, living arrangements apart from the parents, and parenthood. Almost all states allow patients age 18 years or older to give their own consent for a medical procedure; non-emancipated patients younger than age 18 years require the consent of one parent.

2014

29
Q

A 22-year-old woman comes to the office for consultation because she is dissatisfied with the appearance of her nose. History includes two cosmetic procedures of the nose. During the consultation, she also expresses dissatisfaction with the appearance of her eyelids, chin, lower abdomen and flanks, and breast size. In this patient with body dysmorphic disorder, which of the following additional findings is most likely?

A) Anorexia
B) Anxiety disorder
C) Depression
D) Hypochondriasis
E) Substance abuse

A

The correct response is Option C.

The demographic features of body dysmorphic disorder include an onset during late adolescence, and it appears to affect men and women with equal frequency. The clinical features of body dysmorphic disorder most frequently include preoccupation with the appearance of the skin, hair, and nose, although any body part can be a source of concern.

Although several psychiatric comorbidities have been associated with body dysmorphic disorder, depression is the most common. In one study, over 75% of patients with body dysmorphic disorder had a lifetime history of major depression, and over half met criteria for current major depression.

The remaining psychiatric disorders listed also occur with body dysmorphic disorder, but in lower frequencies. Lifetime rates of substance abuse disorders in patients with body dysmorphic disorder have been shown to be close to 30%. Similarly, the lifetime history of an anxiety disorder in patients with body dysmorphic disorder was reported to be more than 6%. The same study reported the lifetime comorbidity rate of anorexia to range from 7 to 14%.

Body dysmorphic disorder and hypochondriasis both involve obsessional thinking and checking behaviors, but the focus of concern in body dysmorphic disorder is on appearance, whereas in hypochondriasis the concerns relate to health status. One study found that only 2% of their body dysmorphic disorder sample had comorbid hypochondriasis.

2014

30
Q

A 38-year-old woman with symptomatic macromastia comes to the office because she desires reduction mammaplasty. A medical assistant obtains the patient’s history, and the plastic surgeon performs a brief history and comprehensive physical examination. The patient is considered a good candidate for surgery, and without further discussion the medical assistant is left to obtain a generic hospital consent form. The patient signs the form without a witness present. The surgeon is at increased medico-legal risk because of which of the following?

A) Lack of a specific plastic surgery consent form
B) Lack of verbal discussion of the risks of reduction mammaplasty
C) Lack of a witness signature on the consent form
D) Use of a medical assistant in the consultation
E) The physician is not at risk because the patient signed a consent form

A

The correct response is Option B.

This surgeon is at increased medico-legal risk due to the fact that there was a lack of verbal discussion of the risks of reduction mammaplasty. Three components must exist for a patient to give informed consent: disclosure, capacity, and voluntariness. In this case, the lack of verbal communication regarding the risks of reduction mammaplasty represents a lack of disclosure. Use of a medical assistant is accepted practice and does not increase the physician’s risk in and of itself. One cannot rely on written documents alone, which may or may not be read or understood by the patient. Although a witness signature is required by many hospitals, the presence of a witness is not a central component of the informed consent process. A specific plastic surgery consent form can be helpful, but only if the risks pertaining to the plastic surgery procedure are discussed in person with the patient. Use of a generic hospital consent form is common and acceptable, provided a verbal discussion of the particular risks associated with the proposed procedure occurs and is documented elsewhere in the medical record.

2014

31
Q

Which of the following individuals is at highest risk of dissatisfaction with the surgical outcome?

A) A 25-year-old man with a large amount of excess skin following a 100-lb (45-kg) weight loss who comes to the office for body contouring
B) A 35-year-old woman who comes to the office for facial rejuvenation surgery in order to advance her career as a news anchor
C) A 42-year-old woman who comes to the office with a large nasal dorsal hump and bulbous tip who is requesting an improved appearance
D) A 45-year-old mother of three with marked deflation and ptosis of the breasts who is looking for an improved appearance
E) A mother of a 3-year-old girl who brings the child in for surgery for a large congenital nevus

A

The correct response is Option B.

One of the psychological contraindications to plastic surgery is when a patient gauges the success of surgery on realization of a specific goal (i.e., a job promotion). Others include the patient who is unable to contemplate an imperfect result, uncertain as to which aspect to change, under emotional stress during consultation, motivated to have surgery at the request of others, and a doctor-shopper dissatisfied with the results of multiple previous procedures.

There are multiple contraindications to surgery: some anatomical and some psychological. One of the most important decisions by a surgeon is whether to perform the requested surgery. The plastic surgeon has to identify a correctable deformity or concern first. This then has to be balanced against the importance that the patient places on this deformity. According to Gorney, the patient with minor deformity but extreme concern is most likely dissatisfied with whatever the outcome. Additionally, the surgical outcome has little to do with the emotional stress.

The dissatisfied patient, once discovered postoperatively, must be handled carefully. This patient must be seen frequently and offered compassion and concern. Allow the patient to see you as the ally that you are. Offer a waiting period before performing any other operations. Consider an offer to revise an operation only if you concur with the patient’s complaints and you think you can improve the appearance. Also, sit with the patient and have a frank discussion of his/her complaints.

All the other options in the question present patients with real identifiable, correctable problems with reasonable expectations.

2014

32
Q

A 67-year-old man undergoes excision of a squamous cell carcinoma from the tip of his nose. The patient’s daughter, who is a physician, asks for the pathology results. Which of the following is the most appropriate response?

A) Ask the patient’s daughter to provide proof of her medical licensure
B) Have the patient’s daughter complete a medical records release form
C) Obtain the patient’s consent to release the results to his daughter
D) Refer the patient’s daughter to the pathology lab
E) Release the pathology results to the patient’s daughter

A

The correct response is Option C.

According to the Health Information Portability and Accountability Act (HIPAA) of 1996, it is a violation to provide personal health information about a patient without the patient’s expressed consent. Consent is ideally documented in the medical record and signed by the patient. According to HIPAA, there are specific Permitted Uses and Disclosures. A physician is “permitted, but not required, to use and disclose protected health information, without an individual’s authorization, for the following purposes or situations: 1) To the Individual (unless required for access or accounting of disclosures); 2) Treatment, Payment, and Health Care Operations; 3) Opportunity to Agree or Object; 4) Incident to an otherwise permitted use and disclosure; 5) Public Interest and Benefit Activities; and 6) Limited Data Set for the purposes of research, public health or health care operations. Covered entities may rely on professional ethics and best judgments in deciding which of these permissive uses and disclosures to make.” Treatment is the provision, coordination, or management of health care and related services for an individual by one or more health care providers, including consultation between providers regarding a patient and referral of a patient by one provider to another.

2014

33
Q

A plastic surgeon flies home after taking her oral American Board of Plastic Surgeons (ABPS) examination. She is excited to finally start advertising to the public in order to grow her practice. She enlists an endorsement from one of her satisfied patients who underwent augmentation mammaplasty. The patient provides a quote and allows her photograph to be used. The plastic surgeon also wants to highlight her additional fellowship training in breast surgery. She is an American Society of Plastic Surgeons (ASPS) candidate member. Inclusion of which of the following items in her advertisement would be a breach of the ASPS Code of Ethics?

A) ASPS logo
B) Assertion of advanced skills in breast surgery as a result of her breast fellowship
C) A list of her plastic surgery training programs
D) Testimonial of the patient stating that this is the best plastic surgeon she knows

A

The correct response is Option A.

The plastic surgeon described is board eligible, not board certified, and a Candidate Member of the ASPS. She cannot use the logo until she is a member of the ASPS, and she must first be board-certified. She cannot assume she has passed her oral board examination until the board contacts her to let her know this. Using the ASPS logo in advertising is a breach of the ASPS Code of Ethics without board certification and full membership in the ASPS.

Claims highlighting the fact that she is a female plastic surgeon to attract female patients do not represent a breach of the Code of Ethics, nor is sharing her educational history. She has enlisted her patient’s endorsement, and use of a direct quotation as a testimonial from the patient is fine if it is clearly a quote from the patient. The plastic surgeon needs to be careful, because other plastic surgeons in her area may think she is breaching the Code of Ethics with claims of superiority, as stated in Section 2IG: The member, either personally or through a partner or associate or any physician or other affiliated health care provider, uses or participates in the use of any form of public or private communication (including computer imaging and electronic communications) containing a false, fraudulent, deceptive, or misleading statement or claim.

2014

34
Q

In a randomized, blinded, placebo controlled trial, 84% of patients in the migraine surgery vs. 58% of patients in the sham surgery group had greater than 50% reduction in migraine symptoms (p < 0.05). Which of the following is indicated by a p value of <0.05?

A) The observed difference is likely due to sampling variation (accept null hypothesis)
B) The observed difference is likely due to sampling variation (reject null hypothesis)
C) The observed difference is not likely due to sampling variation (accept null hypothesis)
D) The observed difference is not likely due to sampling variation (reject null hypothesis)
E) Cannot make a determination regarding the null hypothesis

A

The correct response is Option D.

The p value is defined as the probability of getting a difference at least as large as that observed if the null hypothesis is true. The larger the p value, the more likely the observed difference is due to sampling error (and therefore one accepts the null hypothesis of no difference). The smaller the p value, the more likely the observed difference is not due to sampling error (and therefore one rejects the null hypothesis of no difference).

In patients who suffer from moderate to severe migraine headaches from a single or predominant trigger site, 84% of patients that underwent surgical decompression of that trigger point experienced reduction in migraine symptoms by more than 50%, compared to 58% of those who underwent sham surgery.

2019

35
Q

Which of the following forms of communication is compliant with the Health Insurance Portability and Accountability Act (HIPAA)?

A) E-mailing the confidential information using an encrypted patient portal server
B) Leaving protected information on the patient’s voicemail
C) Placing a sealed folder with patient records under the attending physician’s office door
D) Texting medical information to a password protected smart phone
E) Transferring the patient records via a non-encrypted flash drive

A

The correct response is Option A.

The Health Insurance Portability and Accountability Act of 1996 (HIPAA) is United States legislation that provides data privacy and security provisions for safeguarding medical information. Unauthorized release of any confidential or identifying information, which can be linked to an individual patient, is considered a violation of the law, with penalties ranging from fines to incarceration. The secure, private transmission of Protected Health Information (PHI) is allowed between two treating health-care professionals, provided that the communication is confidential and not at significant risk of breach or theft. Transmission of PHI via social media, e-mail, and other electronic methods must be done through a combination of safeguards that involves encryption. Although the legal understanding of how to communicate PHI continues to evolve, these devices must meet institutional requirements for security.

2019

36
Q

Which of the following scenarios represents a medical “near miss” event?

A) A patient consented for a right carpal tunnel release is surgically prepped for a left carpal tunnel release
B) A patient describes breast firmness and asymmetry three months after implant augmentation
C) A patient develops an asymptomatic pneumothorax after central venous catheter placement
D) A patient prescribed hydroxyzine is treated with hydralazine
E) A patient with a penicillin antibiotic allergy is treated with a cephalosporin antibiotic

A

The correct response is Option A.

This patient has the potential to suffer a wrong site surgery if time-out protocols were not established. The surgical preparation error is a near miss. A “near miss” is an unplanned event that does not result in injury, illness, or damage, but has the potential to do so.

It is within the standard of care to treat patients who report penicillin antibiotic allergies with a cephalosporin antibiotic. Patient with a true penicillin allergy have about a 10% cross-reactivity with cephalosporin antibiotics. Treating a patient prescribed hydroxyzine with hydralazine is a look-alike, sound-alike medication error. Pneumothorax after central venous catheter placement is a complication of the procedure. Breast implant contracture is an inherent risk of breast implant augmentation.

2020

37
Q

A plastic surgery intern is reviewing patient’s charts for the week’s upcoming surgical cases on her service. She realizes that a 55-year-old immediate breast reconstruction patient with diabetes and hypertension, scheduled for surgery in 4 days, was not scheduled for an anesthesia preoperative evaluation and, therefore, had no laboratory studies or electrocardiography scheduled. The intern informs the attending physicians, schedules the appointment, and then calls the patient. She and the scheduler then implement a system by which all patients age 50 years or older who have pre-existing medical problems get an anesthesia preoperative evaluation. This is an example of which of the following core competencies?

A) Interpersonal and communication skills
B) Medical knowledge
C) Patient care and procedural skills
D) Professionalism
E) Systems-based practice

A

The correct response is Option E.

This is an example of systems-based practice. According to the Accreditation Council of Graduate Medical Education (ACGME) milestones, system-based practice subcompetencies include the ability to:

Work effectively in various health care delivery settings and systems relevant to their clinical specialty,

Coordinate patient care within the health care system relevant to their clinical specialty,

Incorporate considerations of cost awareness and risk/benefit analysis in patient care,

Advocate for quality patient care and optimal patient care systems,

Work in interprofessional teams to enhance patient safety and improve patient care quality,

Participate in identifying systems errors and in implementing potential systems solutions.

In this scenario, the intern is demonstrating the ability to “coordinate patient care within the health care system relevant to their clinical specialty” and “participate in identifying systems errors and in implementing potential systems solutions.” The other core competencies are not as relevant as systems-based practice in this scenario.

2020

38
Q

A multi-institutional clinical trial is gathering data on the ability of a test to determine the number of women who develop a new breast disease, and comparing this with age-matched controls. The specificity of the test is defined as which of the following?

A) The ratio of healthy subjects diagnosed as negative and the total number of healthy patients
B) The ratio of healthy subjects diagnosed as positive and the total number of sick patients
C) The ratio of sick patients diagnosed as negative and the total number of healthy patients
D) The ratio of sick patients diagnosed as negative and the total number of sick patients
E) The ratio of sick patients diagnosed as positive and the total number of sick patients

A

The correct response is Option A.

The sensitivity of a test is defined as the ability of a test to correctly classify an individual as diseased (positive in disease).

Sensitivity = a / a + c

The specificity of a test is the ability of a test to correctly classify an individual as disease free.

Specificity = d / b + d

2020

39
Q

A 24-year-old man who is hearing impaired requests to be evaluated for left cubital tunnel syndrome. The patient currently lives 2 hours away, and his mother, who usually helps interpret for him, is unable to attend the appointment. The office does not have anyone who is capable of interpreting sign language. Which of the following is the most appropriate next step?

A) Arrange a video interpreter to be available during the appointment at the office’s expense
B) Decline to schedule an appointment because the office does not offer the language services requested
C) Help the patient arrange for an interpreter to be present at the appointment at his expense
D) Require the patient bring a friend or family member to help interpret during the appointment

A

The correct response is Option A.

The Americans with Disabilities Act (ADA) requires that reasonable accommodations are provided by businesses and in public areas to allow people with disabilities to participate in daily activities. Public places include doctors’ offices. The building and spaces should accommodate all individuals regardless of disability. As a business and a public space, a doctor’s office must be in compliance with the ADA. Services cannot be denied to a patient with a disability because of the disability if services could otherwise be provided. Accommodations should be made to examine the patient with a disability as any other.

In this case, treatment for a cubital tunnel syndrome, something offered routinely by this office, cannot be declined based on the patient’s hearing deficit. In this case of a patient with a hearing issue, an interpreter must be provided to help with the appointment upon the patient’s request. This does not necessarily need to be in-person; a video interpreter can be acceptable. The patient cannot be charged for the interpreter services whether in-person or by video. The health care provider is expected to make a reasonable effort to provide the service. While having the patient bring a friend or family member to the appointment would make the appointment easier for the physician, a patient is not required to bring anyone to help interpret for him/her.

2020

40
Q

In a single physician private practice establishment, which of the following descriptions of a Privacy Official meets the US Department of Health & Human Services (HHS) expectations as delineated in the Health Insurance Portability and Accountability Act (HIPAA)?

A) A group of employees who function as a privacy board
B) An individual who functions as the privacy official and office manager for the practice
C) An outside consultant with expertise in HIPAA-related privacy policy
D) A student intern who functions solely as the privacy official for the practice

A

The correct response is Option B.

The Health Insurance Information Portability and Accountability Act (HIPAA)—also known as the Kennedy-Kassebaum Act—was passed into law in 1996 during the Clinton Administration by the 104th US Congress. HIPAA provides a framework for the security of private health information and how it may be used and shared by health care providers, researchers, payers, and assistants to these entities. There is some scalability built into the law so that large entities such as regional health maintenance organizations (HMOs) and hospital systems have different expectations of how they will accomplish the rules governing privacy compared to a private practice.

According to the U.S. Department of Health and Human Services on its website:

“Responsible health care providers and businesses already take many of the kinds of steps required by the Rule to protect patients’ privacy. Covered entities of all types and sizes are required to comply with the Privacy Rule. To ease the burden of complying with the new requirements, the Privacy Rule gives needed flexibility for providers and plans to create their own privacy procedures, tailored to fit their size and needs. The scalability of the Rule provides a more efficient and appropriate means of safeguarding protected health information than would any single standard.

For example, the privacy official at a small physician practice may be the office manager, who will have other non-privacy related duties; the privacy official at a large health plan may be a full-time position, and may have the regular support and advice of a privacy staff or board. The training requirement may be satisfied by a small physician practice’s providing each new member of the workforce with a copy of its privacy policies and documenting that new members have reviewed the policies; whereas a large health plan may provide training through live instruction, video presentations, or interactive software programs.

The policies and procedures of small providers may be more limited under the Rule than those of a large hospital or health plan, based on the volume of health information maintained and the number of interactions with those within and outside of the health care system.”

2020

41
Q

Which of the following is consistent with the recommendations of The Joint Commission and the Centers for Medicare and Medicaid Services regarding practitioners’ orders and patient-related communication?

A) Computerized provider order entry (CPOE) is not an acceptable method for order submission as it allows providers to directly enter orders into the electronic health record (EHR)
B) Health care organizations should allow the use of unsecured text messaging—that is, short message service (SMS) text messaging from a personal mobile device—for communicating protected health information
C) HIPAA compliance is not maintained if all the information is de-identified before it is transmitted
D) The transmission of a verbal order requires real-time, synchronous clarification and confirmation of the order as it is given by the ordering practitioner

A

The correct response is Option D.

In collaboration with the Centers for Medicare & Medicaid Services (CMS), The Joint Commission developed the following recommendations:

All health care organizations should have policies prohibiting the use of unsecured text messaging—that is, short message service (SMS) text messaging from a personal mobile device—for communicating protected health information. Organizations are expected to incorporate limitations on the use of unsecured text messaging in their policies protecting the privacy of health information. This policy should be routinely discussed during orientation of all practitioners and staff working in the facility.

Computerized provider order entry (CPOE) should be the preferred method for submitting orders as it allows providers to directly enter orders into the electronic health record (EHR). CPOE helps ensure accuracy and allows the provider to view and respond to clinical decision support (CDS) recommendations and alerts.

In the event that a CPOE or written order cannot be submitted, a verbal order is acceptable and it should allow for a real-time, synchronous clarification and confirmation of the order as it is given by the ordering practitioner.

HIPAA compliance can also be maintained by deidentifying information before it is transmitted. Under the Safe Harbor Method, health information is no longer linked to an individual when 18 types of patient identifiers have been removed.

2020

42
Q

A 68-year-old woman presents with multiple injuries sustained during a motor vehicle collision that require reconstructive surgery. She will require a stay in a skilled nursing facility following her initial hospitalization. Which of the following parts of Medicare covers this service?

A) Medicare Part A
B) Medicare Part B
C) Medicare Part C
D) Medicare Part D

A

The correct response is Option A.

Medicare Part A covers services and supplies considered medically necessary to treat a disease. These services include inpatient hospital care, skilled nursing facility care, and hospice environments. In addition, when appropriate, home health services are covered by Part A.

Medicare Part B covers medically necessary services and preventative care services. Physician services, durable medical equipment, and mental health services are included in Part B coverage.

Medicare Part C is also known as Medicare Advantage. Part C allows for a Medicare-eligible individual to select an approved private health insurance plan. Medicare Part D offers prescription drug coverage to original Medicare. Part D can be added to a Medicare Advantage Plan if prescription drug coverage is not included.

2020

43
Q

A study is conducted to evaluate the association between diabetes and postoperative infection in patients undergoing implant-based breast reconstruction with acellular dermal matrix. Which of the following statistical tests is most appropriate to supply the data for this study?

A) Analysis of variance (ANOVA)
B) Linear regression analysis
C) Pearson’s chi-squared test
D) Unpaired T test
E) Wilcoxon rank-sum test

A

The correct response is Option C.

Selecting an appropriate statistical test is critical for accurate data analysis. Determining the optimal method for a given data set must take into account several factors including the limitations and distributional properties of the variables under study.

Statistical variables may be defined as either categorical or numerical. Categorical variables typically represent qualitative observations (eg, postoperative infection, diabetes, obesity) while numerical variables refer to quantitative observations (eg, body mass index, HgbA1c). Additionally, it is important to distinguish between independent (predictive) and dependent (predicted) variables. These variables can also be categorical or numerical. Dependent variables are typically the measured endpoints of the study (eg, postoperative infections – categorical versus operative times – numerical) while independent variables are hypothesized to have an influence over the measured endpoints (eg, diabetes/obesity – categorical versus HgbA1c / BMI – numerical).

Studies, such as this one examining only categorical variables (diabetes and postoperative infection), are best analyzed using Pearson’s chi-squared test. In contrast, a study evaluating only numerical variables is best analyzed using regression analysis.

The unpaired T test and analysis of variance (ANOVA) are best used as statistical tests to analyze independent numerical and dependent categorical data. The tables shown help to provide a general outline for statistical test selection based on the different types of statistical variables being studied including categorical or numerical variables, independent or dependent variables, number of groups being studied, and whether the variables are normally distributed or not.

These statistical tests make assumptions of the parameters of the population distribution and are considered parametric tests. Non-parametric tests, including the Wilcoxon rank-sum and Kruskal-Wallis tests, are used when the data does not meet the assumptions required for parametric tests.

2020

44
Q

A 37-year-old woman comes to the office desiring breast augmentation mammaplasty, mastopexy, and abdominoplasty. The surgeon is trying to promote “mommy-makeover” procedures and asks the patient if she would be willing to participate in a video recording of her operation to be posted on social media. Which of the following statements is correct regarding the informed consent process when obtaining and posting a video to social media that includes protected health information and reveals patient identity?

A) ASPS guidelines recommend that surgeons should not participate in the posting of sensitive content via social media because of the inherent risks to patient privacy
B) A detailed consent documenting the dynamic nature of individual social media sites should be formally discussed and documented in the patient’s medical record prior to proceeding
C) Standard hospital or surgery center consent can be modified to include language about social media and should release the physician for unrestricted use of content on any platform
D) Verbal consent should be obtained the day of the operation and must be witnessed by a nurse

A

The correct response is Option B.

The most correct answer is that a detailed consent documenting the dynamic nature of individual social media sites should be formally discussed and documented in the patient’s medical record prior to proceeding.

In the systematic review and ethical analysis of current plastic surgery publications regarding posting of online video content, Dorfman et al. describe in detail their recommendations for posting sensitive content online.

Although there are no consensus guidelines documented in the American Society of Plastic Surgeons (ASPS) Code of Ethics, social media continues to evolve as an important part of a plastic surgery practice, as more patients report searching online to find their physicians. Authors document five ethical principles to follow when posting content online in order to “protect patients, surgeons, and the public perception of our specialty:”

Ask the patients about posting the content online and obtain a formal written consent. Full disclosure with the patient must involve specific social media sites and that the patient will have the ability to withdraw consent at any time. Legal advice may improve the quality of the consent form.

In understanding the dynamic between the physician and patient and possibility for coercion, the patient must be made aware that they may refuse consent without any punishment, penalty, and delivery of an inferior product, i.e., worse operative result.

The patient must be made aware of the dynamic nature of social media platforms, and must be fully aware that their videos will become public and permanent at the time of publishing online. Withdrawal of consent does not equate to removal of online content because even if the surgeon removes the video, it may persist online indefinitely.

Always follow the standards of professionalism published by the ASPS Code of Ethics.

The surgeon is ultimately responsible for all content disseminated online.

2021

45
Q

A surgeon is tasked with designing a prospective research study investigating whether there is any causality between breast implants and the development of breast implant illness (BII). The surgeon plans to study a group of women with implants and a group without implants and determine whether they develop BII. Which of the following best describes this type of study?

A) Case series study
B) Case-control study
C) Comparative cohort study
D) Cross-sectional study
E) Randomized controlled trial

A

The correct response is Option C.

A comparative cohort study investigates a particular exposure (implants or no implants) to determine correlation to a disease (breast implant illness [BII]).

A case-control study retrospectively identifies cases (BII) and controls (no BII) from the same source population (women) to investigate differences in exposures or risk factors (implants or no implants).

A randomized controlled trial randomly assigns participants into an experimental or control group.

A cross-sectional study is an observational study that analyzes data from a population at a specific time point.

A case series study is one that tracks participants who have received an exposure and tracks outcomes.

2021

46
Q

A recent graduate of an ACGME-accredited plastic surgery residency has set up a private practice for herself. She has applied for and received hospital privileges, state licensure, and malpractice insurance, and has started seeing patients in her clinic and is advertising her practice. In terms of advertising, which of the following aligns with the guidelines and values set forth by the ABPS?

A) ABPS does not have established guidelines
B) A candidate for Board Certification with the ABPS is responsible for all advertising, including third-party employers or physician rating websites
C) Active practice websites are permitted prior to graduation of residency
D) Alcoholic beverages are permitted at marketing events where injectables, procedures, or operations are performed
E) Candidates are permitted to advertise as board eligible for 10 years

A

The correct response is Option B.

In keeping with the values established and published by the ABPS in the Booklet of Information, candidates for board certification with the ABPS are responsible for all advertising, including third-party employers or physician rating websites. All other answers do not align with the values set by the ABPS.

2021

47
Q

A physician is considering different locations to perform a specific medical procedure. According to the Centers for Medicare & Medicaid Services, a higher number of relative value units would be attained by performing this procedure in which of the following places of service?

A) Ambulatory surgical center
B) Community mental health center
C) Inpatient hospital
D) Outpatient hospital
E) Physician’s office

A

The correct response is Option E.

A physician will obtain a higher number of relative value units (RVUs) for a specific medical procedure by performing it in a “non-facility” or “office” setting, such as a physician’s office, compared with performing it in a “facility” setting (hospital, ambulatory surgical center, community mental health center, etc.), according to the fee schedule by the Centers for Medicare & Medicaid Services (CMS).

Physician services are described by CPT codes and Healthcare Common Procedure Coding System codes. CMS determines the number of RVUs assigned for each physician service by adding three subcategories of RVUs:

Physician Work RVUs: reflect the relative time and intensity associated with furnishing a specific medical service. It may reflect not only the “intra-service” time, but also the time needed to prepare for the service beforehand and to document it afterwards.

Practice Expense RVUs: reflect the costs of maintaining a practice (such as renting office space, buying supplies and equipment, and staff costs).

Malpractice RVUs: reflect the costs of medical liability insurance.

When a physician provides a service in a facility (eg, hospital, ambulatory surgical center), the costs of clinical personnel, equipment, and supplies are incurred by the facility, not the physician’s practice. Therefore, CMS assigns to these services a “facility-based” Practice Expense RVU amount that excludes the practice expenses and is typically lower than the “office-based” Practice Expense RVUs for the same service.

CMS uses a formula to determine payment amounts for each covered medical service. First, each of the three RVU subcategories is multiplied by the corresponding geographic practice cost indices, which are designed to account for geographic variations in the costs of practicing medicine in different areas of the country. Then the three adjusted RVU subcategories are added together and multiplied by a conversion factor in dollars.

2021

48
Q

The ABPS Continuous Certification Program in Plastic Surgery includes yearly self-assessment and assessment of knowledge and skills. Which of the following additional criteria need to be met in order to maintain certification?

A) 300 continuing medical education (CME) credits within 3 years
B) Oral board exam after 5 years of practice
C) Verification of unrestricted state medical license
D) Written exam every 10 years

A

The correct response is Option C.

The ABPS Continuous Certification Program, previously known as MOC-PS, includes continuous yearly self-assessment and learning throughout the certification period(s). The requirements include four basic components: (1) professionalism, (2) life long learning and self-assessment, (3) assessment of knowledge, judgment, and skills, and (4) improvement in medical practice. Therefore, an unrestricted state medical license is mandatory, and 150 continuing medical education credits on a 3-year cycle are needed.

Of note, medical specialty certification in the United States is a voluntary process.

2021

49
Q

Which of the following is most likely to be specified in a durable power of attorney document?

A) Agreement to donate organs and tissue
B) Appointment of a health care proxy
C) Do not resuscitate (DNR) orders
D) Permission for mechanical ventilation
E) Withholding of artificial nutrition and hydration

A

The correct response is Option B.

An advance directive allows a person to express his or her desires related to end-of-life medical care. There are two types of advance directive: power of attorney and living will. A durable power of attorney is a document which allows a person to appoint an individual, known as a health care proxy or agent, who can make decisions regarding medical care for that person, in the event that he or she is no longer able to make those decisions for himself or herself. A living will, on the other hand, is a document that outlines a person’s values and preferences for life-sustaining treatments. These may include use of particular equipment (mechanical ventilator or dialysis machine), instructions in the use of CPR, preferences regarding artificial hydration and nutrition (such as tube feeding or withholding nutrition), palliative or comfort care, and tissue and organ donation wishes.

2021

50
Q

A study compares a new injectable neuromodulator drug against placebo for the treatment of glabellar lines. The authors of the study report that if this drug performs no differently than placebo, there is a 0.4% chance that repeating the study will show the same or greater calculated differences between the two groups. Which of the following terms refers to this concept?

A) Alpha
B) p-value
C) Power
D) Type I error
E) Type II error

A

The correct response is Option B.

The p-value is a calculated value that quantifies the probability of obtaining data equal to or more extreme than the data observed on a study, should the null hypothesis be true (eg, the new drug in reality is NOT more efficacious than placebo).

Type I error is the erroneous rejection of a true null hypothesis (eg, a study shows that a new drug is more efficacious than placebo, when in reality it is not).

Alpha is the probability of making a type I error (rejecting a true null hypothesis). It is an assigned value determined by the researcher. A value of 5% is often chosen in medical literature.

Type II error is the failure to reject a false null hypothesis (eg, a study shows that a new drug is no different than placebo, when in reality it is more efficacious). Beta is an assigned value by the researcher that represents the probability of making a type II error. Power of a test of statistical significance is the probability that it will reject a false null hypothesis. It decreases as beta increases (power = 1–beta).

Power of a test may be influenced by multiple factors, including sample size and magnitude of the measured effect.

2021

51
Q

A 34-year-old woman presents for evaluation for a large abdominal pannus, frequent rashes, and skin breakdown as a result of excess skin. BMI is 41 kg/m2. While in the waiting room awaiting consultation, the patient picks up patient education pamphlets on abdominoplasty and panniculectomy. According to the National Institutes of Health and the American Medical Association recommendations, readability of patient education materials should not exceed the reading level of which of the following grades?

A) Fourth
B) Sixth
C) Eighth
D) Tenth
E) High school graduate

A

The correct response is Option B.

Health literacy is defined as “the capacity to obtain, interpret, and understand basic health information and services and the competence to use such information and services to enhance health.” Low health literacy is associated with a poor understanding of personal disease, worse overall health, and increased hospitalizations. Most adults read between the eighth and ninth grade level, but as many as 20% of adults read at the lowest reading level, which is approximately fifth grade.

Several studies have shown that patient education materials are routinely written at a grade level above the recommendations by the National Institutes of Health (NIH) and the American Medical Association (AMA). In this vignette, patient literature written at a reading level that is too advanced may contribute to misunderstanding between the reconstructive nature of a panniculectomy and the cosmetic nature of abdominoplasty.

2022

52
Q

Which of the following standard tools is most appropriate for reporting systematic reviews and meta-analyses that assess the benefits and harms of health care intervention?

A) Cochrane
B) Introduction, Methods, Results, and Discussion (IMRaD)
C) Preferred Reporting Items for Systematic Reviews and Meta-analyses (PRISMA)
D) Quality of Reporting Meta-analyses (QUOROM)
E) Standard Protocol Items: Recommendations for Interventional Trials (SPIRIT)

A

The correct response is Option C.

The Preferred Reporting Items for Systematic Reviews and Meta-analyses (PRISMA) statement has been published and endorsed by many high-impact medical journals as the standard tool to be used for systematic review and meta-analysis. The PRISMA standard supersedes the Quality of Reporting of Meta-analyses (QUOROM) standards. The Cochrane is a charitable group that conducts systematic reviews of health care interventions and diagnostic tests and publishes them in the Cochrane Library. Standard Protocol Items: Recommendations for Interventional Trials (SPIRIT) provides evidence-based recommendations for the minimum content of a clinical trial protocol. The Introduction, Methods, Results, and Discussion (IMRaD) format refers to standard structure for scientific writing.

2022

53
Q

Plastic surgery, like all surgical specialties, demonstrates a lack of racial and ethnic diversity in its workforce, including its resident workforce. According to the American Association of Medical Colleges’ definition of racial and ethnic under-representation in the medical profession, which of the following is the most appropriate description of the plastic surgery resident workforce as of 2017?

A) Asian people are proportionally represented
B) Black people are underrepresented
C) Latinx people are proportionally represented
D) White, non-Latinx people are overrepresented

A

The correct response is Option B.

Since 2000, the white residency population has been appropriately represented, meaning commensurate with the population as a whole in the United States. The Latinx and Black populations are represented at about half of their population in the United States. The Asian populations are represented at four to five times their population in the United States. The Pacific Islanders, Native/First Peoples, and Mixed Races percentages are too small to make meaningful analysis about whether they are appropriately represented or under-represented.

2022

54
Q

A plastic surgery residency program director is developing criteria for allowing postgraduate year 1 and 2 residents to perform laceration repairs in the emergency department without direct supervision by a more senior resident. In addition to at least 2 months on a plastic surgery rotation and 10 repairs observed and coached directly by a senior practitioner, the program director decides that the resident must submit a video of the resident repairing a standard laceration on a facial laceration model, which will be graded against a published rubric. In this scenario, which of the following is the purpose of the simulation of a facial laceration repair?

A) Certification to practice
B) Deliberate practice
C) Technique training
D) Verification of proficiency

A

The correct response is Option D.

For psychomotor skills like surgical skills, simulations can be used for training, practice, assessment, and certification. When simulation is part of a curriculum, its purpose must be clear to all stakeholders: the trainee, the instructor, the program, and the public.

Following adult learning principles, simulation used for training and instruction should be done in a low-stakes situation where the trainee has opportunity to try to a new skill without worrying about causing harm to another or being judged. For more basic skills with inexperienced individuals, low-fidelity experiences can inexpensively provide large gains in proficiency. As the complexity of the skill and the sophistication of the trainee increase, higher-fidelity experiences become more useful.

Deliberate practice is used to improve performance. Deliberate practice is directed at a specific aspect of performance and is observed and coached at the time of practice, such as having a golf professional coach by watching and redirecting performance of a player after each drive. Practicing by oneself is not deliberate practice. Self-directed practice has the potential to habituate and propagate mistakes. The repetition with immediate feedback by a mentor allows correct habits to develop and extinguishes bad habits.

Simulation can be used to verify that a trainee has learned the steps of a basic task such as laceration repair. The simulation can include the entire task (anesthetizing, cleansing, draping, repairing, and dressing) or can be limited to any step of the test. This model works well when the learner has an opportunity for observed practice and unobserved practice. It allows an element of self-evaluation as the resident has the rubric and submits the video when they feel they’ve achieved the criteria.

Simulation is being incorporated into surgical certification rapidly. A prominent example is the requirement to have passed Fundamentals of Laparoscopy Surgery and Fundamentals of Endoscopic Surgery during residency as prerequisite for the board certification process by the American Board of Surgery.

2023

55
Q

Which of the following healthcare reform laws guarantees the transferability of a patient’s health insurance coverage from one employer to another?

A) 21st Century Cures Act
B) Health Information Technology for Economic and Clinical Health Act
C) Health Insurance Portability and Accountability Act
D) Patient Safety and Quality Improvement Act
E) Women’s Health and Cancer Rights Act

A

The correct response is Option C.

Numerous healthcare reform acts have been recently enacted as legislation. Many of these involved improved patient access to care as well as providing additional benefits to patient safety and privacy. While the Health Insurance Portability and Accountability Act (HIPAA) was the first to institute regulations for protected health information through its privacy rules, this legislation also guarantees the transferability of a patient’s healthcare insurance coverage from one job to another.

The Patient Safety and Quality Improvement Act protects healthcare workers who report unsafe conditions and encourages reporting of medical errors. The Health Information Technology for Economic and Clinical Health Act promotes the expansion and adoption of health information technology while updating both the privacy rules as well as an individual’s right to access their electronic health information (EHR). The 21st Century Cures Act helps define the specifics of EHR interoperability and increases a patient’s access to a broadened scope of their medical record in a timely fashion. The Women’s Health and Cancer Rights Act mandates insurance coverage of breast reconstruction after cancer treatment.

2023

56
Q

Data collection is complete in a study examining Patient-Reported Outcomes Measurement Information System Upper Extremity (PROMIS UE) scores following intramedullary screw placement in the treatment of an isolated proximal phalanx fracture. Which of the following is the most appropriate statistical test to compare the mean PROMIS UE scores at 2 weeks with those at 12 weeks?

A) Independent samples t-test
B) One-sample t-test
C) One-way analysis of variance
D) Paired samples t-test

A

The correct response is Option D.

Understanding appropriate statistical analysis is essential for conducting and reviewing research. There are four main ways in which means can be compared, assuming that the data are normally distributed.

An independent samples t-test is a comparison of the mean for two different data sets that are independent from each other. For example, an independent samples t-test might be used to compare PROMIS UE scores at the end of healing between male and female patients, where male versus female is the independent variable.

A paired samples t-test is a comparison of two measurements on the same subject at two different time points. The paired sample t-test is appropriate in this scenario because researchers are looking for differences in PROMIS UE scores at two discrete times after surgery.

A one-sample t-test is used when a mean is compared with a specified constant, such as comparing PROMIS UE scores at one time point with the expected PROMIS UE score in the general adult population.

One-way analysis of variance is used to compare the means in more than two groups.

2023

57
Q

In the design of a proper randomized clinical trial, concerns about a selection bias confounder are best addressed at which of the following stages?

A) Data analysis
B) Data collection
C) Data publication
D) Initial literature search
E) Study design

A

The correct response is Option E.

Confounders are extraneous risk factors associated with both causal factors and disease related entities that can influence accurate data analysis. These can result in overestimation or underestimation of the study effect. The five main parts of a well-designed clinical trial study are the initial literature review, study design and execution, data collection, data analysis, and data publication. Examples of confounders that may negatively impact these specific stages include publication bias (initial literature review), selection bias (study design and execution), and information bias (data collection).

2023

58
Q

A 57-year-old woman comes to the office to discuss breast reconstruction after mastectomy. When considering the dimensions of access to healthcare, the distance between the patient’s geographic location and her reconstructive surgeon’s location most significantly affects which of the following?

A) Accessibility
B) Accommodation
C) Affordability
D) Availability
E) Awareness

A

The correct response is Option A.

The results of Penchansky and Thomas’s 1981 findings have been modified and summarized to explain modern concerns for access to care as follows:

  1. Availability – relationship of the volume and type of existing services to the client’s
    volume and type of needs.
  2. Accessibility – relationship between the location of supply and the location of the client.
  3. Accommodation – relationship between the manner in which the supply resources are
    organized and the client’s ability to accommodate those factors.
  4. Affordability – Relationship of the prices of services and the provider’s insurance to the
    client’s income, ability to pay, and existing health insurance.
  5. Awareness – Awareness of services and the indications for its use.

2023

59
Q

A plastic surgeon evaluates an 18-year-old woman for potential gluteal augmentation with high-volume fat grafting. The patient has seen the surgeon’s posts on social media, and she states that she wants the same results but is unable to afford the cosmetic fees. The surgeon offers a discount on her procedure if she allows him to livestream the operation. She agrees, and the operation is performed the following week. Which of the following best describes the ethical violation in this interaction?

A) Age criterion not met for surgical consent
B) Discount offered by the plastic surgeon
C) Patient’s lack of control over the posted video
D) Sexual harassment
E) None; both parties consented to the agreement

A

The correct response is Option B.

The American Society of Plastic Surgeons Code of Ethics states that there can be no coercion (such as financial coercion) on the part of a plastic surgeon to get a patient to participate in online marketing. Sexual harassment is defined as unwanted sexual advances, requests for sexual favors, and other verbal or physical harassment. For a competent adult, 18 years is considered the age of consent for self-treatment. Depending on the agreement with the social media venue used for posting, both the patient and the surgeon may lose future control over the video, but this is not an ethical violation.

2023

60
Q

Which of the following best describes the type and level of evidence in a Plastic and Reconstructive Surgery paper evaluating a retrospective series of surgical outcomes in 13 patients over 8 months?

A) Diagnostic, Level II
B) Diagnostic, Level III
C) Diagnostic, Level IV
D) Therapeutic, Level II
E) Therapeutic, Level III
F) Therapeutic, Level IV

A

The correct response is Option F.

The paper would fall into the therapeutic category, not diagnostic or risk, because the authors are discussing a surgical intervention. It is a case study looking at a result and would therefore be considered level IV evidence.

In early 2011, Plastic and Reconstructive Surgery unveiled their new program of a specialty wide evidence-based initiative. From that time forward, level of evidence had to be listed on any and all articles that were amenable to such grading. This was believed to be a straightforward and visible way of promoting and advancing evidence-based medicine in the practice of plastic surgery. The three types of clinical questions addressed are diagnostic, therapeutic, and risk. Each article should fall into one of the three groups.

There are five levels of evidence, ranging from I being the most stringent, highest degree of evidence to V being mostly expert opinion-based. Level I studies are high-quality, multicenter or single-center, randomized controlled trials with adequate power or systematic reviews of these trials. Level II are lesser-quality randomized controlled trials or systematic reviews of these studies. Level III are retrospective cohort or case-controlled studies. Level IV are case studies with pre/post-test or only post-test outcomes. Level V studies encompass expert opinion developed by consensus, case reports, and clinical examples. The long-term goal of this process is to elevate the level of evidence of articles in Plastic and Reconstructive Surgery.

2023

61
Q

According to the American Board of Plastic Surgery, the certification of a diplomate who fails to maintain the required ethical or professionalism standards may be classified under which of the following statuses?

A) Expired
B) Probation
C) Retired
D) Revoked
E) Suspended

A

The correct response is Option D.

According to the American Board of Plastic Surgery, a certified diplomate has fulfilled three requirements: 1) completed the written exam, 2) completed the oral exam, and 3) met the residency training requirements in an accredited institution. In the years following, certification status may change for a variety of reasons:

  • Expired – Certification has lapsed; the diplomate has not met the requirements to renew certification.
  • Retired – No longer in active practice of plastic surgery.
  • Revoked – Certification is no longer valid due to a failure to maintain required ethical or professionalism standards.
  • Suspended – Certification is suspended due to action on a medical license by a state medical board.
  • Probation – Certification is placed on probation due to action on a medical license by a state medical board.

2023